More Interest Formulas: EGN 3615 Engineering Economics With Social and Global Implications

You might also like

Download as pptx, pdf, or txt
Download as pptx, pdf, or txt
You are on page 1of 66

Chapter 4

More Interest Formulas


EGN 3615
ENGINEERING ECONOMICS
WITH SOCIAL AND GLOBAL
IMPLICATIONS

1
Chapter Contents
Uniform Series Compound Interest Formulas
Uniform Series Compound Amount Factor
Uniform Series Sinking Fund Factor
Uniform Series Capital Recovery Factor
Uniform Series Present Worth Factor
Arithmetic Gradient
Geometric Gradient
Nominal Effective Interest
Continuous Compounding

2
Uniform Series Compound Amount Factor
A A A A F1+F2+F3+F4 =F

0 1 2 3 4 0 1 2 3 4
A F1

0 1 2 3 4 F1  A (1  i )3
A F2

0 1 2 3 4 F2  A (1  i ) 2

A F3

0 1 2 3 4 F3  A (1  i )
A=F4

0 1 2 3 4
F4  A
3
Uniform Series Compound Amount Factor
That is, for 4 periods,

F = F1 + F 2 + F 3 + F 4

= A(1+i)3 + A(1+i)2 + A(1+i) + A

= A[(1+i)3 + (1+i)2 + (1+i) + 1]

4
Uniform Series Compound Amount Factor
F
For n periods with interest (per period),
A A A A A A

0 1 2 3 4 n-1 n

F = F1 + F2 + F3 + … + Fn-1 + Fn

= A(1+i)n-1 + A(1+i)n-2 + A(1+i)n-3 + … + A(1+i) + A

= A[(1+i)n-1 + (1+i)n-1 + (1+i)n-3 + …+ (1+i) + 1]

5
Uniform Series Compound Amount Factor

 (1  i ) n  1
F A   A ( F / A, i, n)
 i 
Notation
Uniform Series
Compound Amount Factor
i = interest rate per period
n = total # of periods
6
Uniform Series Formulas (Compare to slide 25)
(1) Uniform series compound amount: Given A, i, & n, find F
F = A{[(1+i)n – 1]/i} = A(F/A, i, n) (4-4)
(2) Uniform series sinking fund: Given F, i, & n, find A
A = F{i/[(1+i)n – 1]} = F(A/F, i, n) (4-5)
(3) Given F, A, & i, find n
n = log(1+Fi/A)/log(1+i)
(4) Given F, A, & n, find i
There is no closed form formula to use.
But rate(nper, pmt, pv, fv, type, guess)
7
Uniform Series Compound Amount Factor
 Question: If starting at EOY1, five annual deposits of $100 each are
made in the bank account, how much money will be in the account
at EOY5, if interest rate is 5% per year?
F= 552.6
i=0.05
0 1 2 3 4 5

$100 $100 $100 $100 $100


 (1  i ) n  1  (1  0.05) 5  1
F A   100    100(5.526)
 i   0.05 
8
QUESTION CONTINUES (USING INTEREST TABLE)
F= 552.6
i=0.05
0 1 2 3 4 5

$100 $100 $100 $100 $100

 (1  i ) n  1
F A   A ( F / A, i %, n)  100 (5.526)
 i 

9
QUESTION CONTINUES(SPREADSHEET)

A= ($100) $552.6
i= 5%
n= 5
Go to XL
--Chap 4 extended examples-A1
Use function: FV(rate, nper, pmt, pv, type)
10
Uniform Series Compound Amount Factor
 Question: Five annual deposits of $100 each are made into an
account starting today. If interest rate is 5%, how much money
will be in the account at EOY5?
F1 F2= 580.2
i=5%
0 1 2 3 4 5

$100 $100 $100 $100 $100


 (1  i ) n  1  (1  0.05) 5  1
F1  A    100    100 (5.526)  552.6
 i   0.05 
F2  F1 (1  i ) n  552.6 (1  0.05)1 11
QUESTION CONTINUES(INTEREST TABLE)
F1 F2= 580.2
i=5%
0 1 2 3 4 5

$100 $100 $100 $100 $100


 (1  i ) n  1
F1  A    A ( F / A, i %, n)
 i 
 A ( F / A, 5%, 5)
 100 (5.526)  552.6
F2  F1 ( F / P, 5%, 1)  552.6 (1.050) 12
QUESTION CONTINUES(SPREADSHEET)

A= ($100) $552.6
i= 5% $580.2
n= 5

13
Uniform Series Compound Amount Factor
 Question: If starting at EOY1, five annual deposits of $100 each are
made in the bank account, how much money will be in the account
at EOY5, if interest rate is 6.5% per year?
F= ?
i=6.5%
0 1 2 3 4 5

$100 $100 $100 $100 $100


( F / A,6.5%,5)  ?
( F / A,6%,5)  5.6371
( F / A,7%,5)  5.7507
14
( F / A,6%,5)  5.6371
INTERPOLATION ( F / A,7%,5)  5.7507

6.0 5.6371
0.5 X
1 6.5 0.1136

7.0 5.7507
0. 5 X

1 0.1136
X  0.0568
(F/A, 6.5%, 5)  5.6371  0.0568  5.6939
F  100(F/A, 6.5%, 5)  100 (5.6939)  569.39
Interpolation
15
Uniform Series Sinking Fund Factor
F=Given
i=Given
0 1 2 3 4 5
n=Given

A=?

A= Equal Annual Dollar Payments


F= Future Some of Money
i = Interest Rate Per Period
n= Number of Interest Periods
16
Uniform Series Sinking Fund Factor

 i 
AF   F (A / F , i %, n)
 (1  i )  1
n

Notation
Uniform Series
Sinking Fund
Factor

17
Uniform Series Sinking Fund Factor
Question: A family wishes to have $12,000 in a bank account
by the EOY 5. to accomplish this goal, five annual deposits
starting at the EOF year 1 are to be made into a bank account
paying 6% interest. what annual deposit must be made to
reach the stated goal?
F=$12,000
i=5%
0 1 2 3 4 5
n=5

A =$2172
 i   0.05 
AF   12,000    12,000 (0.1810)
 (1  i )  1  (1  0.05)  1
n 5
18
QUESTION CONTINUES(INTEREST TABLE)
F=$12,000
i=5%
0 1 2 3 4 5
n=5

A = $2172
 i 
AF   F ( A/F, i%, n)
 (1  i )  1
n

 F ( A/F, 5%, 5)
 12,000 (0.1810)
19
Uniform Series Sinking Fund Factor
 Question: A family wishes to have $12,000 in a bank account by the
EOY 5. to accomplish this goal, six annual deposits starting today
are to be made into a bank account paying 5% interest. What
annual deposit must be made to reach the stated goal?
F=$12,000
i=5%
0 1 2 3 4 5
n=6

A = $1764
 i   0.05 
AF   12,000    12,000 (0.1470)
 (1  i )  1  (1  0.05)  1
n 6
20
$1764
QUESTION CONTINUES(INTEREST TABLE)
F=$12,000
i=5%
0 1 2 3 4 5
n=6

A = $1764
 i 
AF   F ( A/F, i%, n)
 (1  i )  1
n

 F ( A/F, 5%, 6)
 12,000 (0.1470)
21
Uniform Series Sinking Fund Factor
 Example: the current balance of a bank account is $2,500. starting
EOY 1 six equal annual deposits are to be made into the account.
The goal is to have a balance of $9000 by the EOY 6. if interest rate
is 6%, what annual deposit must be made to reach the stated goal?
F=$9000
i=6%
0 1 2 3 4 5 6

P=$2,500 A = $796.23
FPvalue  2,500 (F/P, 6%, 6)  2,500 (1.419)  3547.50
FTotal  9000  3547.50  5452.50
A  5452.50 (A/F, 6%, 6)
A  5452.50 (0.1434) 22
Uniform Series Capital Recovery Factor
P=Given
i=Given
0 1 2 3 4 5
n

A=?
P= Present Sum of Money
A= Equal Annual Dollar Payments
i = Interest Rate
n= Number of Interest Periods
23
Uniform Series Capital Recovery Factor
 i  n i 
AF   P(1  i )  
 (1  i )  1  (1  i )  1
n n

 i (1  i ) 
n
AP   P (A / P, i %, n)
 (1  i )  1
n

Notation
Uniform Series
Capital
Recovery Factor 24
Uniform Series Formulas (Compare to slide 7)
(1) Uniform series present worth: Given A, i, & n, find P
P = A{[(1+i)n – 1]/[i(1+i)n]} = A(P/A, i, n) (4-7)
(2) Uniform series capital recovery: Given P, i, & n, find A
A = P{[i(1+i)n]/[(1+i)n – 1]} = P(A/P, i, n) (4-6)
(3) Given P, A, & i, find n
n = log[A/(A-Pi)]/log(1+i)
(4) Given P, A, & n, find i (interest/period)
There is no closed form formula to use.
But rate(nper, pmt, pv, fv, type, guess)
25
Uniform Series Capital Recovery Factor
 Example: A person borrows $100,000 from a commercial bank. The
loan is to be repaid with five equal annual payments. If interest rate
is 10%, what should the annual payments be?
P= $100,000
i=10%
0 1 2 3 4 5

A =26,380
 i (1  i ) n   0.1(1  0.1) 5 
AP   100,000  
 (1  i )  1  (1  0.1)  1
n 5

 100,000 (0.2638)
26
Example CONTINUES(INTEREST TABLE)
P= $100,000
i=10%
0 1 2 3 4 5

A =26,380

A  P ( A/P, i%, n)
 100,000 ( A/P, 10%, 5)
 100,000 (0.2638)
27
Uniform Series Capital Recovery (MS EXCEL)
Use function: PMT(rate, nper, pv, fv, type)
rate = interest rate/period
nper = # of periods
pv = present worth
fv = balance at end of period n (blank means 0).
type = 1 (payment at beginning of each period) or
0 (payment at end of a period)(blank means 0)

See spreadsheet

28
Uniform Series Capital Recovery Factor
Example: At age 30, a person begins putting $2,500 a
year into account paying 10% interest. The last deposit
is made on the man’s 54th birthday (25 deposits).
Starting at age 55, 15 equal annual withdrawals are
made. How much should each withdrawal be?
Solution
Step 1: First A will be converted into F.
Step2: F will be considered as P.
Step3: P will be converted into Second A

29
EXAMPLE CONTINUES
F = 245,868
i=10%
0 1 2 3 21 22 23 24

A=$2500
F  2500 ( F/A, 10%, 25)  2500 (98.347)
i=10%

0 1 2 3 12 13 14 15

A =$32,332
P= $245,868
A  245,868 ( A/P, 10%, 15)  245,868 (0.1315)
30
Uniform Series Present Worth Factor

A=Given

0 1 2 3 4 5
n=Given
i=Given
P=?

A= Equal Annual Dollar Payments


P= Present Sum of Money (at Time 0)
i = Interest Rate/Period
n= Number of Interest Periods
31
Uniform Series Present Worth Factor

 (1  i ) n  1
PA n 
 A ( P / A, i, n)
 i (1  i ) 

Notation
Uniform Series
Present Worth
Factor

32
Uniform Series Present Worth Factor
 Example: A special bank account is to be set up. Each year, starting
at EOY 1, a $26,380 withdrawal is to be made. After five withdrawals
the account is to be depleted. if interest rate is 10%, how much
money should be deposited today?
A=26,380

0 1 2 3 4 5

i=10%
P= 100,001
 (1  i ) n  1  (1  0.1) 5  1
PA n 
 26,380  5
 26,380(3.791)
 i (1  i )   0.1(1  0.1)  33
EXAMPLE CONTINUES (USING INTEREST TABLE)

A=26,380

0 1 2 3 4 5

i=10%
P= 100,001
P  A ( P/A, i%, n)
 26,380 ( P/A, 10%, 5)
 26,380 (3.791)
34
Uniform Series Present Worth (Using MS EXCEL)
Use function: PV(rate, nper, pmt, fv, type)
rate = interest rate/period
nper = total # of periods (payments)
pmt = constant payment/period
fv = balance at end of period n (blank means 0)
type = 1 or 0
PV(0.1, 5, -26380) = $100,000.95
See spreadsheet

35
Uniform Series Present Worth Factor
Example: Eight annual deposits of $500 each are made into a
bank account beginning today. Up to EOY 4, the interest rate
is 5%. After that, the interest rate is 8%. What is the present
worth of these deposits?

PTotal  ?
i=5% i=8%

0 1 2 3 4 5 6 7

P3 P2 A=500 P1
36
EXAMPLE CONTINUES
PTotal
i=5% i=8%

0 1 2 3 4 5 6 7

P3 P2 A=500 P1
P3  500 P2  500(P/A, 5%, 4) P1  500 (P/A, 8%, 3)
 500(3.546)  1773  (P/F, 5%, 4)
 500(2.577)(0.8227)
PTotal  P1  P2  P3  3333.05
 1060.05
37
EXAMPLE CONTINUES (Using MS EXCEL)
P1 = PV(0.08, 3, -500)(1+0.05)–4
= (1288.55)(0.8227)
= $1,060.09

P2 = PV(0.05, 4, -500)
= $1,772.98

PTotal  P1  P2  P3  $3,333.07

38
Arithmetic Gradient
Arithmetic Gradient series (G): each annual amount differs
from the previous one by a fixed amount G.
A+4G
A+3G
A+2G
A+G A A A A A
A

0 1 2 3 4 5 = 0 1 2 3 4 5

+
P  A( P / A, i, n) 2G
3G 4G
G
 G ( P / G , i , n) 0
0
1 2 3 4 5

39
Arithmetic Gradient Present Worth Factor
Given G, i, & n, find P (4-19)

 (1  i ) n  in  1
PG 2   G ( P / G, i %, n)
 i (1  i )
n

Notation
Arithmetic Gradient
Present Worth Factor

40
Arithmetic Gradient Present Worth Factor
 Question: You has purchased a new car. the following maintenance
costs starting at EOY 2 will occur to pay the maintenance of your car
for the 5 years. EOY2 $30, EOY3 $60, EOY4 $90, EOY5 $120. If
interest rate is 5%, how much money you should deposit into a
bank account today? $120
$30 $60 $90
0
0 1 2 3 4 5 i=5%
G=$30
P= $247.11
 (1  i ) n  in  1  (1  0.05) 5  0.05(5)  1
PG 2   30    30(8.237)
 i (1  i )  0.05 (1  0.05)
n 2 5
  41
QUESTION CONTINUES (INTEREST TABLE)

$90 $120
$30 $60
0
0 1 2 3 4 5 i=5%
G=$30
P= $247.11
P  G (P/G, i%, n)
 30 (P/G, 5%, 5)
 30(8.237)
42
Arithmetic Gradient Present Worth Factor
Question: If interest rate is 8%, what is the present worth of
the following sums?
600
550
500
450 400 400 400 400 400
400

0 1 2 3 4 5 = 0 1 2 3 4 5

+
150 200
100
50
0
0 1 2 3 4 5

43
QUESTION CONTINUES
400 400 400 400 400 ATotal  A1  A2
0 1 2 3 4 5
 400  92
A1  400  492
200 P  492 ( P / A, 8%, 5)
150
100
50  492 (3.9927)
0
0 1 2 3 4 5
 1,964.41
A2  50 ( A / G, 8%, 5)  50 (1.8465)
 92.33  92
44
Arithmetic Gradient Uniform Series Factor

Convert an arithmetic gradient series into a uniform series


Given G, i, & n, find A (4-20)

 (1  i ) n  in  1
AG   G ( A / G, i %, n)
 i (1  i )  i 
n

Notation
Arithmetic Gradient
Uniform Series Factor

45
Arithmetic Gradient Uniform Series Factor
 Question: Demand for a new product will decrease as competitors
enter the market. What is the equivalent annual amount of the
revenue cash flows shown below? (interest 12%)
3000 3000 3000 3000 3000 3000
2500
2000
1500
1000

0 1 2 3 4 5 = 0 1 2 3 4 5

+
AEQIV  3000  500 ( A / G, 12%, 5) 0 1 2 3 4 5
0
 3000  500(1.775) 500
1000
1500 2000
 2112 .50 46
Geometric Series Present Worth Factor
Geometric series: Each annual amount is a fixed percentage
different from the last. In this case, the change is 10%.
We will look at this problem in a few slides.
?
?
?
g=10% ?
?
$133 ?
$100 $110 $121
0 1 2 3 4 5 6 7 8 9 10

i=5%
P=? 47
Geometric Gradient
 Unlike the Arithmetic Gradient where the amount of period-by-
period change is a constant, for the Geometric Gradient, the period-
by-period change is a uniform growth rate (g) or percentage rate.

Year First year maintenance cost Cash Flow


1 100 Uniform growth rate (g)  100  A1
2 100  10%(100)  100(1  0.1)1  110  A 2
3 110  10%(110 )  100(1  0.1) 2  121  A 3
. . . .
. . . .
. . . .
n A n -1  10%( A n -1 )  100(1  g ) n 1  An
48
Geometric Series Present Worth Factor
Given A1, g, i, & n, find P (4-29) & (4-30)

1  (1  g ) n (1  i )  n 
P  A1   where i  g
 ig 

Geometric Series
Present Worth Factor
When Interest rate equals the growth rate,
1
P  A1n(1  i ) where i  g
49
Geometric Series Present Worth Factor
Question: What is the present value (P) of a geometric
series with $100 at EOY1 (A1), 5% interest rate (i), 10%
growth rate (g), and 10 interest periods (n)?
?
?
?
g=10% ?
?
$133 ?
$100 $110 $121
0 1 2 3 4 5 6 7 8 9 10

i=5%
P=? 50
Geometric Series Present Worth Factor
$236
$214
g=10% $195
$177
$133 $146 $161
$110 $121
$100
0 1 2 3 4 5 6 7 8 9 10
i=5%
P= $1184.67
1  (1  g ) n (1  i )  n  1  (1  0.1)10 (1  0.05) 10 
P  A1    100  
 ig   0.05  0.1 
 100(11 .8467) 51
Time for a Joke!
What is Recession?
Recession is when your neighbor loses his or her job.

What is Depression?
Depression is when you lose yours.

By Ronald Reagan

52
Problem 4-7
Purchase a car: $3,000 down payment
$480 payment for 60 months

If interest rate is 12% compounded monthly, at what


purchase price P of a car can one buy?
Solution
i = 12.0%/12 = 1.o% per month, n = 60, and A = $480
P = 3000 + 480(P/A, 0.01, 60)
= 3000 + 480(44.955)
= $24,578

Important: P = $3000 + $480(60) = $31,800, if i = 0.


53
Problem 4-9
$25 million is needed in three years.
Traffic is estimated at 20 million vehicles per year.
At 10% interest, what should be the toll per vehicle?

(a) Toll receipts at end of each year in a lump sum.

(b) Traffic distributed evenly over 12 months, and


toll receipts at end of each month in a lump
sum.

54
Problem 4-9
Solution
(a) Let x = the toll/vehicle. Then
F = $25,000,000 i = 10%/year, n = 3 years
Find A (=20,000,000x).
A = F(A/F, 0.1, 3)
20,000,000x = 25,000,000(0.3021)
x = $0.3776 = $0.38 per vehicle

55
Problem 4-9
Solution
(b) Let x = the toll/vehicle. Then
F = $25M i = (1/12)10%/month, n = 36 months
Find A (=20,000,000x/12).
A = F{i/[(1+i)n–1]}
20,000,000x/12 = 25,000,000{(0.1/12)/(1+0.1/12) 36–
1}
x = $0.359 = $0.36 per vehicle

56
Problem 4-32
If i = 12%, for what value of B is the PW = 0?
Solution
Consider now = time 1. Then
PW = B+800(P/A, 0.12, 3) – B(P/A, 0.12, 2) – B(P/F, 0.12, 3)
= 1921.6 – 1.758B
Letting PW = 0 yields B = $1,093.06

For any cash flow diagram,


if PW = 0, then its worth at anytime = 0!

57
Problem 4-46
Solution
FW = FW
[1000(F/A, i, 10)](F/P, i, 4) = 28000
By try and error:
At i = 12%, LHS = [1000(17.549)](1.574) = $27,622 too low
At i = 15%, LHS = [1000(20.304)](1.749) = $35,512 too high

Using linear interpolation:


i = 12% + 3%[(28000 – 27622)/(35512 – 27622)]
= 12.14%
58
Use of MS EXCEL
pmt(i, n, P, F, type) returns A, given i, n, P, and F

sinking fund (P=0) A = F{i/[(1+i)n – 1]} (4-


5)
capital recovery (F=0) A = P{[i(1+i)n]/[(1+i)n – 1]} (4-6)
or combined (P ≠ 0, and F ≠ 0)

rate(n, A, P, F, type, guess) returns i, given n, A, P, and F

59
Use of MS EXCEL
pv(i, n, A, F, type) returns P, given i, n, A, and F

present worth (A=0) P = F/(1+i)n (3-5)


series present worth (F=0) P = A{[(1+i)n – 1]/[i(1+i)n]} (4-7)
or combined (A ≠ 0, and F ≠ 0)

fv(i, n, A, P, type) returns F, given i, n, A, and P

compound amount (A=0) F = P(1+i)n (3-3)


series compound amount (P=0) F = A{[(1+i)n – 1]/i} (4-4)
or combined (A ≠ 0, and P ≠ 0)

60
Use of MS EXCEL
nper(i, A, P, F, type) returns n, given i, A, P, and F.

If A = 0, n = log(F/P)/log(1+i) single payment


If P = 0, n = log(1+Fi/A)/log(1+i) uniform series
If F = 0, n = log[A/(A-Pi)]/log(1+i) uniform series

effect(r, m) returns ia, given r and m.


effective annual interest rate ia = (1+r/m)m –1 (3-
7)

nominal(ia, m) returns r, given ia and m.


nominal annual interest rate r = m[(ia – 1)1/m + 1] 61
A Real Life Case
Mr. Goodman set up a trust fund of $1.5M for his 2
children in 1991. It is worth more than $300M today
(January 2012).
What is the effective annual interest rate?

Solution
P = $1.5M, F = $300M, n = 20 years
ia = (F/P)1/n – 1 = (300/1.5)1/20 – 1 = 30.332%
ia = rate(20, 0, 1.5, –300) = 30.332%
ia = rate(20, 0, -1.5, 300) = 30.332%
62
End of Chapter 4
Uniform Series Compound Interest Formulas
Uniform Series Compound Amount Factor: F/A
Uniform Series Sinking Fund Factor: A/F
Uniform Series Capital Recovery Factor: A/P
Uniform Series Present Worth Factor: P/A
Arithmetic Gradient
Geometric Gradient
Spreadsheet Solutions

63
Interpolation-1
Given: F(X1); F(X2)
What is F(X3) where X1 < X3 < X2?
Assuming linearity so that a linear equation will do:
Basic equation: y = mx + b so
1. F(X1) = mX1 + b
2. F(X2) = mX2 + b
Subtract 2 from 1:
F(X1)-F(X2) = m (X1-X2)  m = (F(X1)-F(X2))/(X1-X2)
From 1 we get b = (F(X1) - mX1)

64
Interpolation-2
F(X1)-F(X2) = m (X1-X2)  m = (F(X1)-F(X2))/(X1-X2)
From 1 we get b = (F(X1) - mX1)
Now
F(X3) = m X3 + b
= m X3 + F(X1) - mX1 = m (X3 - X1) + F(X1)
= (X3 - X1) (F(X1)-F(X2))/(X1-X2) + F(X1)
= F(X1) + (F(X1)-F(X2)) (X3 - X1) /(X1-X2)

65
Interpolation-3
F(X3) = F(X1) + (F(X1)-F(X2)) (X3 - X1) /(X1-X2)

Suppose that the Xs are interest rates, i, and the Fs are


the functions (F/A,i,n), then
F(i3) = F(i1) + (F(i1)-F(i2)) (i3 - i1) /(i1-i2)

Return
66

You might also like